aboutsummaryrefslogtreecommitdiff
path: root/li/hw3.tex
blob: 870daf0bc7f1dce9277977f10332bd9588a71e5e (plain)
1
2
3
4
5
6
7
8
9
10
11
12
13
14
15
16
17
18
19
20
21
22
23
24
25
26
27
28
29
30
31
32
33
34
35
36
37
38
39
40
41
42
43
44
45
46
47
48
49
50
51
52
53
54
55
56
57
58
59
60
61
62
63
64
65
66
67
68
69
70
71
72
73
74
75
76
77
78
79
80
81
82
83
84
85
86
87
88
89
90
91
92
93
94
95
96
97
98
99
100
101
102
103
104
105
106
107
108
109
110
111
112
113
114
115
116
117
118
119
120
121
122
123
124
125
126
127
128
129
130
131
132
133
134
135
136
137
138
139
140
141
142
143
144
145
146
147
148
149
150
151
152
153
154
155
156
157
158
159
160
161
162
163
164
165
166
167
168
169
170
171
172
173
174
175
176
177
178
179
180
181
182
183
184
185
186
187
188
189
190
191
192
193
194
195
196
197
198
199
200
201
202
203
204
205
206
207
208
209
210
211
212
213
214
215
216
217
218
219
220
221
222
223
224
225
226
227
228
229
230
231
232
233
234
235
236
237
238
239
240
241
242
243
244
245
246
247
248
249
250
251
252
253
254
255
256
257
258
259
260
261
262
263
264
265
266
267
268
269
270
271
272
273
274
275
276
277
278
279
280
281
282
283
284
285
286
287
288
289
290
291
292
293
294
295
296
297
298
299
300
301
302
303
304
305
306
307
308
309
310
311
312
313
314
315
316
317
318
319
320
321
322
323
324
325
326
327
328
329
330
331
332
333
334
335
336
337
338
339
340
341
342
343
344
345
346
347
348
349
350
351
352
353
354
355
356
357
358
359
360
361
362
363
364
365
366
367
368
369
370
371
372
373
374
375
376
377
378
\def\bmatrix#1{\left[\matrix{#1}\right]}

    {\noindent\bf Section 2.2}

{\noindent\bf 12.}

{\it (a)}

This is correct. It's equal to the number of linearly independent
rows/dimension of the row space/rank because if any of the non-zero rows
were linearly dependent, they would have been eliminated to a zero row
when forming $R.$

{\it (b)}

This is false. A zero matrix has rank zero but can have a nonzero number
in this property if it has more columns than rows.

{\it (c)}

This is true. All columns are either pivot columns or free columns, and
the rank is the number of pivot columns.

{\it (d)}

No. The following matrix has four ones but rank one:
$$\bmatrix{1&1&1&1}$$

{\noindent\bf 26.}

The maximum rank of a matrix is the smaller of its number of rows and
its number of columns because the pivot columns and rows are strictly
less than the total number of each. Therefore, $C$ and $A$ have at most
rank 2, and $CA$ also has at most rank 2 (column space of $A$ is a
superset of the column space of $CA,$ which becomes obvious if they're
treated like functions). $CA$ is a $3\times 3$ matrix, and $I_3$ has
rank 3, so $CA \neq I.$

$AC = I$ if
$$A = \bmatrix{1 & 0 & 0\cr
               0 & 1 & 0}$$
               and
$$C = \bmatrix{1 & 0\cr
               0 & 1\cr
               0 & 0}$$

{\noindent\bf 42.}

If $Ax = b$ has infinitely many solutions, then there exists infinitely
many solutions to $Ay = 0$ if $y = x - x_0$ where $x_0$ is a particular
solution to $Ax_0 = b.$ If there exists one particular solution $x_1$ to
$Ax_1 = B,$ then there must be an infinite number $A(x_1+y) = B$ where
$y$ is in the null space of $A$ as noted earlier.

However, $Ax = B$ could have zero solutions. The matrix
$$A = \bmatrix{1&0\cr 0&0}$$
does not include $b_0 = \bmatrix{0\cr 1}$ in its column space, so $Ax =
b_0$ would have zero solutions even though $Ax = \bmatrix{1\cr 0}$ has
an infinite number of solutions.

\iffalse % practice problems

{\noindent\bf 7.}

$$R_3 = R_2 + R_1 \to c = 5 + 2.$$

{\noindent\bf 9.}

{\it (a)}

$$\bmatrix{1&2&3&4\cr 0&0&1&2\cr 0&0&0&0}\bmatrix{x_1\cr x_2\cr x_3\cr
x_4} = \bmatrix{0\cr 0\cr 0} \to x = \bmatrix{-4\cr 0\cr -2\cr 1}x_4 +
\bmatrix{-2\cr 1\cr 0\cr 0}x_2$$
$$R = \bmatrix{1&2&0&-2\cr 0&0&0&1&2\cr 0&0&0&0}.$$
$$Rx = 0 \to x = \bmatrix{2&0&-2&1}x_4 + \bmatrix{-2&1&0&0}x_2.$$

{\it (b)}

If the right-hand side is $(a, b, 0),$ the solution set will be the null
space plus a particular solution. In the case of $U,$ a particular
solution would be $(a, 0, b, 0).$

{\noindent\bf 10.}

$$\bmatrix{0&1&-1\cr 1&0&-1}x = \bmatrix{1\cr -2\cr 0}.$$
$$\bmatrix{0&1&-1\cr 1&0&-1\cr 1&1&-2}x = \bmatrix{1\cr -2\cr 0}.$$

{\noindent\bf 14.}

$$R_A = \bmatrix{1&2&0\cr 0&0&1\cr 0&0&0}.$$
$$R_B = \bmatrix{1&2&0&1&2&0\cr 0&0&1&0&0&1\cr 0&0&0&0&0&0}.$$
$$R_C = \bmatrix{1&2&0&0&0&0\cr 0&0&1&0&0&0\cr 0&0&0&0&0&0\cr
0&0&0&1&2&0\cr 0&0&0&0&0&1\cr 0&0&0&0&0&0}.$$

{\noindent\bf 21.}

The rank $r$ is the number of pivot rows and the number of pivot
columns, so the subset of these rows and columns would be an $r\times r$
matrix. They are by definition linearly independent, so each spans/forms
a basis for $R^r,$ giving them invertibility.

{\noindent\bf 24.}

The rank of $A$ is the same as the rank of $A^T,$ so
$${\rm rank}(AB) \leq {\rm rank}(A) \to {\rm rank}((AB)^T) \leq
{\rm rank}(A^T) \to {\rm rank}(B^TA^T) \leq {\rm rank}(A^T) \to
{\rm rank}(AB) \leq {\rm rank}(B).$$

{\noindent\bf 25.}



{\noindent\bf 36.}

{\it (a)}

All vectors in $R^3$ are in the column space, so only the trivial
combination of the rows of $A$ gives zero.

{\it (b)}

Only vectors where $b_3 = 2b_2$ are within the column space. This means
that $2x_2 = -x_3$ gives a zero combination. % double check this.

{\noindent\bf 40.}

$x_5$ is a free variable, the zero vector isn't the only solution to
$Ax=0,$ and if $Ax=b$ has a solution, then it has infinite solutions.

{\noindent\bf 43.}

{\it (a)}

$q=6$ gives a rank of 1 for $B,$ and $q=3$ gives a rank of 1 for the
frist matrix.

{\it (b)}

$q = 7$ gives a rank of 2 for both matrices.

{\it (c)}

A rank of 3 is impossible for both matrices.

{\noindent\bf 45.}
% idk come back to this.

{\it (a)}

$r < n.$

{\it (b)}

$r > m.$ $r\geq n.$ % ???

{\it (c)}

$r < n.$

{\it (d)}

{\noindent\bf 53.}

{\it (a)}

False. The zero matrix has $n$ free variables.

{\it (b)}

True. If the linear function corresponding to the matrix can be
inverted, it must not have a non-zero null-space (i.e. a non-injective
relation).

{\noindent\bf 60.}

$$\bmatrix{1&0&-2&-3\cr0&1&-2&-1}$$ has this nullspace.

{\noindent\bf 61.}

% simple enough to construct

{\noindent\bf 62.}



{\noindent\bf 63.}
{\noindent\bf 64.}

{\noindent\bf 65.}

$$\bmatrix{0&0\cr 1&0}$$

{\noindent\bf 66.}

Dimension of null space is $n-r = 3-r,$ and dimension of column space is
$r,$ so they cannot have the same dimension and therefore cannot be
equal.

\fi

    {\noindent\bf Section 2.3}

{\noindent\bf 22.}

{\it (a)}

They might not span ${\bf R}^4$ if, for example, they are all the zero
vector, but they could span it, like if the first four were elementary
vectors $e_1$ to $e_4.$

{\it (b)}

They are not linearly independent because 4 is the maximal independent
set.

{\it (c)}

Any four might be a basis for ${\bf R}^4,$ because they could be
linearly independent and four vectors in ${\bf R}^4$ could span it.

{\it (d)}

$Ax = b$ might not have a solution. It could have a solution depending
on the $b,$ but $0x = e_1,$ where $0$ refers to the zero vector for $A$
has zero solutions.

{\noindent\bf 27.}

The column space of $A$ has basis in $\{(1, 0, 1)^T, (3, 1, 3)^T\}$ and
the column space of $U$ has basis in $\{(1, 0, 0)^T, (3, 1, 0)^T\}.$
The two matrices have the same row space, based in $\{(1, 3, 2), (0, 1,
1)\}.$
They also have the same null space, based in $\{(-1, 1, -1)\}.$

{\noindent\bf 32.}

{\it (a)}

The dimension is 3 because this is the set of vectors on ${\bf R}^4$
under one linear constraint: $v_4 = -(v_3 + v_2 + v_1).$

{\it (b)}

The dimension is 0 because the identity matrix, by definition only
returns 0 if given 0.

{\it (c)}

The dimension is 16 because there are 16 unconstrained components.

{\noindent\bf 36.}

6 independent vectors satisfy $Ax=0$ by the rank theorem. $A^T$ has the
same rank, so 53 independent vectors satisfy $A^Ty = 0.$

{\noindent\bf 42.}

$\{x^3, x^2, x, 1\}$ form a basis of the polynomials of degree up to 3,
and this set restricted to those where $p(1) = 0$ has basis $\{x^3-1,
x^2-1, x-1\}.$

\iffalse % practice problems

{\noindent\bf 7.}

$$v_1 - v_2 + v_3 = w_2 - w_3 - w_1 + w_3 + w_1 - w_2 = 0,$$
proving dependence of these vectors.

{\noindent\bf 8.} % not an actual problem

$$c_1v_1 + c_2v_2 + c_3v_3 = c_1(w_2 + w_3) + c_2(w_1 + w_3) + c_3(w_1 +
w_2) = (c_2+c_3)w_1 + (c_1+c_3)w_2 + (c_1+c_2)w_3 = 0.$$
Because the set of $w$ vectors are independent, this sum is only equal
to zero if $c_2 + c_3 = 0 \to c_3 = -c_2,$ $c_1 + c_3 = 0 \to c_1 = -c_3
= +c_2,$ and $c_1+c_2 = 0 \to c_2 = c_1 = 0 \to c_3 = 0.$

{\noindent\bf 9.}

{\it (a)}

If $v_1$ to $v_3$ are linearly independent, the dimension of their
spanning set must be 3 (and the set equal to $R^3$), so $v_4 \in R^3$
can be written as a combination of the other three.

{\it (b)}

$v_2 = kv_1$ where $k\in\bf R$

{\it (c)}

$0v_1 + k(0,0,0) = 0,$ giving a non-trivial combination with the value
0.

{\noindent\bf 12.}

The vector $b$ is in the subspace spanned by the columns of $A$ when
there is a solution to $Ax = b.$ The vector $c$ is in the row space of
$A$ when there is a solution to $A^Tx = c$ or $x^TA = c.$

The zero vector is in every space, so the rows may still be independent.
(False)

{\noindent\bf 13.}

The dimensions of the column spaces and of the row spaces of $A$ and $U$
are the same (2), and the row spaces are the same between the two (and
conversely, the null space)

{\noindent\bf 21.}

% easy

{\noindent\bf 23.}

If they are linearly independent, the rank of $A$ is $n.$ If they span
$R^m,$ the rank is $m.$ If they are a basis for $R^m,$ then both are
true and $n = m.$

{\noindent\bf 25.}

{\it (a)}

The columns are linearly independent, so there is no nontrivial linear
combination equal to 0.

{\it (b)}

The columns of $A$ span $R^5,$ so there must be a linear combination
(value of $x$) equal to $b.$

{\noindent\bf 26.}

{\it (a)}

True. Thm in the book.

{\it (b)}

False. See 31.

{\noindent\bf 31.}

If we let $v_k = e_k,$ the subspace with basis $(0, 0, 1, 1)$ does not
have a basis in the elementary vectors.

{\noindent\bf 34.}

% seems simple enough, don't know if I can do it.

{\noindent\bf 35.}

{\it (a)}

False. The unit vector $e_1$'s single column is linearly independent,
but except in $R,$ it doesn't span $R^k,$ and $e_1x = e_2$ has no
solution.

{\it (b)}

True. The rank is at most $5,$ meaning there must be two free variables.

{\noindent\bf 41.}

{\it (a)}

For dimension 1, $y_k = kx.$

{\it (b)}

For dimension 2, $y_1 = x^2,$ $y_2 = 2x,$ and $y_3 = 3x.$

{\it (c)}

For dimension 3, $y_k = x^k.$

\fi

\bye